LSAT and Law School Admissions Forum

Get expert LSAT preparation and law school admissions advice from PowerScore Test Preparation.

 Administrator
PowerScore Staff
  • PowerScore Staff
  • Posts: 8937
  • Joined: Feb 02, 2011
|
#47094
Complete Question Explanation
(The complete setup for this game can be found here: lsat/viewtopic.php?t=11700)

The correct answer choice is (E)

Note the careful selection of the last three days in the question stem condition. As established in the game discussion, the last three days have a special characteristic, namely that G, S, and one other service will be scheduled in some order for the last three days. Answer choices (A), (B), (C), and (D) each contain both G and S, and so they at least meet the basic requirement for the placement of G and S. Answer choice (E), on the other hand, contains G but not S. This means that S would have to be scheduled for one of the first three days, a violation of the second and third rules. Thus, answer choice (E) cannot be true and is correct.

Answer choice (A): Because all of the incorrect answers could be possible, this answer choice can be used to confirm that answer choice (A) in question #3 is incorrect because it reveals that T would be scheduled for an earlier day than L (if G, S, and L are the last three services scheduled, then T would have to be on for the first three services scheduled).

The following full hypothetical proves that this answer choice could be true, and is therefore incorrect:
D10_game #1_#4_diagram 1.png
Answer choice (B): The following full hypothetical proves that this answer choice could be true, and is therefore incorrect:
D10_game #1_#4_diagram 2.png
Answer choice (C): This answer choice can be used to confirm that answer choice (B) in question #3 is incorrect because it reveals that L would be scheduled for an earlier day than P.

The following full hypothetical proves that this answer choice could be true, and is therefore incorrect:
D10_game #1_#4_diagram 3.png
Answer choice (D): This answer choice can be used to confirm that answer choice (D) in question #3 is incorrect because it reveals that W would be scheduled for an earlier day than T.

The following full hypothetical proves that this answer choice could be true, and is therefore incorrect:
D10_game #1_#4_diagram 4.png
Answer choice (E): This is the correct answer choice. As explained above, this is the correct answer.
You do not have the required permissions to view the files attached to this post.

Get the most out of your LSAT Prep Plus subscription.

Analyze and track your performance with our Testing and Analytics Package.